Đến nội dung

Kamii0909 nội dung

Có 155 mục bởi Kamii0909 (Tìm giới hạn từ 21-04-2020)



Sắp theo                Sắp xếp  

#668675 Chứng minh di chuyển trên 1 đườnng cố định

Đã gửi bởi Kamii0909 on 17-01-2017 - 17:10 trong Hình học

Cách của mình. Ta sẽ nhắc lại không chứng minh một số kết quả cơ bản sau.
1. Cho tam giác $ABC$ nội tiếp $(O)$. Tiếp tuyến tại $B,C$ của $(O)$ cắt nhau tại $P$. Khi đó $AP$ là đường đối trung đỉnh $A$ của tam giác $ABC$.
2. Trong tam giác $ABC$, đường đối trung đỉnh $A$ cắt $BC$ tại $D$. Khi đó $\frac{DB}{DC}= \frac{AB^2}{AC^2}$
Để xử lí cho gọn, đẹp trước hết ta đi chứng minh bổ đề sau
$\textbf{Bổ đề}$ Cho tam giác $ABC$. Đường đối trung đỉnh $A$ cắt $BC$ tại $Q$. $O,I,J$ lần lượt tâm đường tròn ngoại tiếp tam giác $ABC,AQB,AQC$. Khi đó $OA$ chia đôi $IJ$.
$\textbf{Chứng minh}$
Kẻ $AH,AM$ là đường cao và là trung tuyến của tam giác $ABC$.
Dễ thấy phép vị tự quay tâm $A$ biến $\Delta AIJ \rightarrow \Delta ABC$ biến $H \rightarrow Q$. Mặt khác cũng có $\angle HAM= \angle QAO$ nên biến $ AM \rightarrow AO$. Lại có $AM$ là trung tuyến trong tam giác $ABC$ nên $AO$ là trung tuyến trong tam giác $AIJ$.

$\textbf{Quay lại bài toán}$
Qua $Q$ kẻ đường thẳng $\parallel BC$ cắt $AB,AC$ tại $E,F$. Gọi $I,J$ là tâm đường tròn ngoại tiếp các tam giác $AQE,AQF$.
Theo bổ đề $OA$ chia đôi $IJ$. Ta sẽ chứng minh rằng $O_{1}I=O_{2}J$.
Thật vậy, có $\Delta AIO_{1} \sim \Delta QEB, \Delta AJO_{2} \sim \Delta QFC$ nên ta thu được biến đổi sau.
$$\dfrac{IO_{1}}{JO_{2}}= \frac{IO_{1}}{IA}\cdot \frac{IA}{JA} \cdot \frac{JA}{JO_{2}} = \frac{EB}{QE}\cdot \frac{AB}{AC}\cdot \frac{FQ}{FC}= \frac{AE^2}{AF^2} \cdot \frac{QF}{QE}=1$$
Vậy ta có điều phải chứng minh.



#667825 Chứng minh A,F,I thẳng hàng

Đã gửi bởi Kamii0909 on 09-01-2017 - 22:40 trong Hình học

Nó quanh quanh cấu hình bài hình của USAMO 2008



#667463 $BM,CN,PD$ đồng quy

Đã gửi bởi Kamii0909 on 07-01-2017 - 15:24 trong Hình học

Cho tam giác $ABC$ có đường tròn nội tiếp $(I)$ tiếp xúc $BC,CA,AB$ tại $D,E,F$. Đường tròn $(PBC)$ tiếp xúc $(I)$ tại $P$. Gọi $M,N$ là trung điểm $DE,DF$. Chứng minh rằng $PD,BM,CN$ đồng quy.



#667457 $F,M,Y,Z$ đồng viên

Đã gửi bởi Kamii0909 on 07-01-2017 - 14:37 trong Hình học

Bài này không khó, chắc lấy ý tưởng từ bài Trường Đông năm nay.
$\textbf{Lời giải}$
Kẻ $CH$ cắt $(O)$ tại $K$. $FX$ cắt $AK$ tại $N$.
Dễ thấy $FX=FN$ nên áp dụng định lý con bướm đảo vào tứ giác nội tiếp $AKBC$ tâm $O$ có $OF \perp FX$.
Phần sau dễ rồi.



#667078 Đề Thi VMO năm 2017

Đã gửi bởi Kamii0909 on 05-01-2017 - 13:37 trong Thi HSG Quốc gia và Quốc tế

Cách của e cho b hình. Hơi dài.
Dễ thấy $RHEF$ điều hòa và $RH \parallel EF$ nên $RS$ chia đôi $EF$. Ta cmr $BP,CQ$ chia đôi $EF$.
Gọi $K$ là giao điểm $BP,CQ$. $G$ là giao điểm $AD,EF$.
Theo định lý Pascal cho 6 điểm $A,D,B,C,P,Q$ có $\overline{K,E,F}$.
Mặt khác dễ thấy $BFEC$ nội tiếp nên $BFGD,CEGD$ nội tiếp.
Có $\angle{BGC}=\angle{BGD}+\angle{CGD}=\angle{BFD}+\angle{CED}=\angle{BKC}$ nên $B,K,G,C$ đồng viên.
Gọi $X$ là giao $BC,EF$.
Có $XB.XC=XE.XF=XG.XK$ mà $(EF,XG)=-1$ nên $K$ là trung điểm $EF$. Ta có đpcm.



#667003 $\frac{1}{a^{2}}+\frac{1...

Đã gửi bởi Kamii0909 on 04-01-2017 - 23:21 trong Bất đẳng thức và cực trị

Đặt $(a,b,c) \rightarrow \left( \dfrac{1}{x},\dfrac{1}{y},\dfrac{1}{z} \right)$
Ta biến đổi bđt cần cm về
$$x^2+y^2+z^2+3 \geq 2(\frac{1}{x}+\frac{1}{y}+\frac{1}{z})\Leftrightarrow x^2+y^2+z^2+2xyz+1 \geq 2(xy+yz+xz)$$
Bất đẳng thức cuối quen thuộc.



#666998 $\frac{a^{2}}{c}+\frac{b^...

Đã gửi bởi Kamii0909 on 04-01-2017 - 23:15 trong Bất đẳng thức và cực trị

làm gì có a + b + c = 1 hả bạn !!

Có đấy bạn. Nếu không có chọn $a=b=c$ có ngay điều vô lý.



#666905 $\frac{a^{2}}{c}+\frac{b^...

Đã gửi bởi Kamii0909 on 04-01-2017 - 12:34 trong Bất đẳng thức và cực trị

Ta có $$\sum \frac{a^2}{c}= \sum \frac{a^4}{a^2c} \geq \frac{(\sum a^2)^2}{\sum a^2c}$$
Ta phải chứng minh
$$\sum a^2 \geq 3(\sum a^2c) \Leftrightarrow (\sum a^2)(\sum a) \geq 3(\sum a^2c) \Leftrightarrow \sum a(a-b)^2 \geq 0$$



#666871 $\sum \dfrac{abc+b+c-a}{a^2+1} \geq...

Đã gửi bởi Kamii0909 on 03-01-2017 - 22:23 trong Bất đẳng thức và cực trị

Với $a,b,c \geq 0$ chứng minh rằng
$\dfrac{abc+b+c-a}{a^2+1}+\dfrac{abc+c+a-b}{b^2+1} +\dfrac{abc+a+b-c}{c^2+1} \geq a+b+c$



#665755 $QF$ tiếp xúc với $(CFM)$

Đã gửi bởi Kamii0909 on 24-12-2016 - 20:45 trong Hình học

nếu có $GC=GF$ ta suy ra $\widehat{CGM}=180^o-2\widehat{BFC}=90^o ,$ , từ $M$ kể tiếp tuyến $Mx$ của $(G)$ thì có hàng $M(xNBC=-1$ , suy ra $FQ$ tiếp xúc $(G)$ . Bây giờ ta đy chứng min $GF=GC$ , ta có$\frac{DF}{FC}=\frac{Sin\widehat{DBE}}{Sin\widehat{EBC}}=\frac{DE}{DB}.\frac{BC}{EC}=\frac{FO}{DG}$ suy ra tam giác $ODF$ đồng dạng $GCF$ suy ra tam giác $GFC$ cân tại $G$ suy ra $GF=GC$ suy ra dpcm

Bạn chỉ rõ cho mình tại sao $\frac{BC}{EC}.\frac{DE}{DB}=\frac{FO}{DG}$ được không? Bạn viết hơi tắt khúc này.



#665740 $P=\frac{x}{x^{2}+1}+\frac{...

Đã gửi bởi Kamii0909 on 24-12-2016 - 18:00 trong Bất đẳng thức - Cực trị

Brazil MO 2008 nhé bạn.



#665696 $QF$ tiếp xúc với $(CFM)$

Đã gửi bởi Kamii0909 on 23-12-2016 - 23:54 trong Hình học

Cho hình vuông $ABCD$ nội tiếp $(O)$ và điểm $E$ trên cạnh $CD$. $AE$ cắt $BC$ tại $G$. $BE$ cắt $(O)$ tại $F$. Lấy $M \neq F$ trên $BE$ sao cho $GM=GF$. Gọi $N$ là trung điểm $BC$. $MN$ cắt $CD$ tại $Q$. Chứng minh rằng $QF$ tiếp xúc với $(CFM)$.



#664998 $2^{x}=a^{b}+1$

Đã gửi bởi Kamii0909 on 18-12-2016 - 15:17 trong Số học

Với $x=1$ dễ có $a=1$ và $b$ nguyên dương bất kì là nghiệm.
Xét $x \geq 2$
Khi đó $4|L.H.S$ nên dễ có $a,b$ lẻ.
Do $b$ lẻ nên $a+1|a^b+1=2^x$
Do đó $a=2^k -1$($k$ nguyên dương)
Dễ xử lí TH $k=1$.
Xét $k \geq 2$. Khi đó $4|(2^k-1)-(-1)$
Ta có $v_{2}{[(2^k-1)^b+1]}=v_{2}{[(2^k-1)^b-(-1)^b]}\\ =v_{2}{(2^k-1+1)}+v_{2}{(b)}=k=v_{2}{(x)}=x$
Vậy $k=x$ nên $b=1$
Nghiệm là $(a,b,x)=(2^t-1,1,t)$



#664447 Tìm bộ số thỏa mãn

Đã gửi bởi Kamii0909 on 12-12-2016 - 11:28 trong Số học

Ta có
$m^2+n^2|(m^2+n^2)(m+n)-mn(m+n)-4$ nên $m^2+n^2|mn(m+n)+4$(1)
Từ đó $m^2+n^2 \leq m^2n +n^2m+4$(2)
Nếu cả 2 số $m,n$ cùng bằng 1 thì (1) thỏa mãn và $m^2+n^2=2$ cũng thỏa mãn.
Ngược lại, KMTTQ giả sử $m>1$
Ta viết lại (2) như sau
$n^2(m-1)+nm^2-m^2+4 \leq 0$
Xét $\Delta$ theo $n$ thì
$\Delta = m^4+4m^3-4m^2-16m+16 \leq 0$ Rõ ràng bất phương trình này vô nghiệm nguyên dương với $m>1$.
Vậy $m=n=1$ là nghiệm duy nhất.
Xử lí theo cách này có thể không dùng điều kiện nguyên tố.



#664444 Tìm các số nguyên tố p thỏa:$p^2+23$ có đúng 6 ước chung.

Đã gửi bởi Kamii0909 on 12-12-2016 - 11:10 trong Số học

Nếu $p=2$ thì $A=p^2+23=27(KTM)$. Nếu $p=3$ thì $A=32(TM)$
Nếu $p>3$ thì $4,3|p^2-1$ nên $4,3|p^2+23$. Dễ thấy $p$ sẽ có ít nhất các ước $1,2,3,4,6,12,p^2+23$(vô lý)
Vậy $p=3$ là giá trị duy nhất.



#664443 Tìm số tự nhiên n>1 nhỏ nhất để $(n+1)(2n+1)\vdots 6$ và t...

Đã gửi bởi Kamii0909 on 12-12-2016 - 11:02 trong Số học

Đóng góp 1 cách cho bài 2.
Giả sử tồn tại số nguyên tố $p$ sao cho $p=a^n +b^n +c^n +d^n$. Do $ab=cd$ nên ta viết lại
$p= a^n +b^n +c^n +\frac{a^n.b^n}{c^n} \\ \Leftrightarrow p.c^n=(c^n+a^n)(c^n+b^n)$
Nên 1 trong 2 số $c^n+a^n,c^n+b^n$ phải là bội của $p$ mà rõ ràng 2 số này $<p$ nên mâu thuẫn. Vậy ta có đpcm.



#664362 Tìm $x,y,z\in N^{*}$ sao cho $xyz=x^{2...

Đã gửi bởi Kamii0909 on 11-12-2016 - 13:20 trong Số học

Biến đổi về dạng
$z=\frac{x^2+3}{xy+2}$ hay $xy+2|x^2+3 \Rightarrow x^2+3 \geq xy+2 // \Leftrightarrow x^2-xy+1 \geq 0$
Coi đây là phương trình bậc 2 ẩn $x$ thì phải có $\Delta =y^2-4 \leq 0$
Vậy $y=1,2$



#664361 $\boldsymbol{Topic}$ Các bài toán số học HSG Toán 8 + 9

Đã gửi bởi Kamii0909 on 11-12-2016 - 13:14 trong Số học


ĐỀ BÀI

$\boxed{1}$: (Hellenic Mathematical Competitions 2013)
Xác định tất cả các bộ ba số nguyên dương (x, y, z) thỏa mãn phương trình sau đây:


$\frac{1}{x}+\frac{2}{y}-\frac{4}{z}=1$

$\boxed{2}$: (Hellenic Mathematical Competitions 2013) Xác định tất cả các số nguyên x và y thỏa mãn phương trình sau đây:


$y=2x^2+5xy+3y^2$

$\boxed{3}$ Tìm nghiệm nguyên của phương trình


$x^3+y^3=(x+y)^2$

Bài 1.
Biến đổi tương đương ta có
$x=\frac{yz}{yz+4y-2z}$
Nên $yz+4y-2z|yz$.
Từ đó có $z \geq 2y$
Nếu $z=2y$ ta có bộ $(1,t,2t)$ thỏa mãn.
Xét $z>2y$
Lại có $yz+4y-2z|2z-4y$ nên $(y-4)(z+8) \leq 32$(*)
Nếu $y \geq 6$ thì $z \geq 12$.
Khi đó dễ thấy (*) vô lý.
Vậy $y \leq 5$
Đến đây dễ rồi.

Bài 2.
Xét $\Delta$ theo $x$ ta có
$\Delta = y^2+8y=(y+4)^2-16=a^2$ với $a$ là số tự nhiên
$\Leftrightarrow (y+4-a)(y+4+a)=16$

Bài 3.
Dễ thấy có nghiệm $(x,y)=(t,-t)$
Xét TH $x+y$ khác $0$.
Biến đổi pt về dạng $x^2-xy+y^2=x+y$
$\Leftrightarrow x^2-x(y+1)+y^2-y=0$
Coi đây là phương trình bậc 2 ẩn $x$ có $\Delta = -3y^2+6y+1 \geq 0$
$\Leftrightarrow 3y^2-6y-1 \leq 0$
Bất phương trình có nghiệm nguyên $y=0,1,2$.
Thế vào ta có $x$.



#664004 Xác định vị trí của điểm $M$ để biểu thức: $P=\frac{...

Đã gửi bởi Kamii0909 on 06-12-2016 - 20:35 trong Hình học

Bài 1.
Xét TH $M$ thuộc cung nhỏ $AD$. Các TH còn lại chứng minh tương tự.
Lấy $G$ trên $AC$ sao cho $\widehat{BMC}=\widehat{AMG}$
Dễ có $\Delta BMC \sim \Delta AMG$ và $\Delta AMB \sim GMC$
Từ đó $\frac{AC}{ME}=\frac{AG}{ME} +\frac{GC}{ME}= \frac{BC}{MD}+ \frac{AB}{MF}$
Từ đó $P=\frac{2AC}{ME}$.
Dễ thấy $P$ không tồn tại GTNN.
Ở đây GTLN $P$ khi $MA=MC$.



#663924 Điều kiên đồng quy của 3 đường thẳng

Đã gửi bởi Kamii0909 on 05-12-2016 - 22:53 trong Hình học

Mình nghĩ hình như đề bài bạn đưa ra chưa chính xác, vì khi mình kiểm tra bằng geogebra thì tứ giác $ABNM$ không phải lúc nào cũng điều hòa. Bạn có thể xem hình mình gửi kèm thì $BM$ không đi qua cực của $AN$ với $(O).$ Mình thử làm và phát hiện ra 1 số tứ giác điều hòa (khác $ABNM$ ), bạn xem thử nhé. Mình chỉ làm chiều thuận thôi, chiều đảo tương tự.

Spoiler

Rất xin lỗi bạn vì nhầm đề. Thực sự thì mình không còn đề gốc,tuy nhiên sau khi suy luận(vì mình đã từng đọc đáp án bài này) có lẽ nó là như sau: Ta xác định các điểm $A,B,C,D,G,P,S,O$ như trên. $PS$ cắt $OD$ tại $Q$. Qua $Q$ kẻ đường thẳng $d$ song song $AD$. Gọi $M,N$ là giao điểm của $d$ và $QP$ với $(O)$.Chứng minh rằng $AG,PS,BD$ đồng quy khi và chỉ khi $MBNG$ điều hòa.
Bài toán trên thực sự rất đơn giản. Mình không nghĩ đề HSG lại quá đánh đố học sinh như bài gốc. Dù sao, mình cũng rất cảm ơn bạn đã chỉ ra sai sót,mình sẽ cố gắng đi tìm bài toán chính xác nhanh nhất có thể.



#663699 Chứng minh CD vuông góc OE

Đã gửi bởi Kamii0909 on 03-12-2016 - 14:48 trong Hình học

Còn 1 cách nữa suy nghĩ thêm đi :D

Cách khác.
Ta nhắc lại không chứng minh bổ đề quen thuộc sau.
Cho tứ giác nội tiếp $ABCD$ có 2 tiếp tuyến tại $B,D$ và $AC$ đồng quy. Khi đó 2 tiếp tuyến tại $A,C$ và $BD$ cũng đồng quy.(Tứ giác điều hòa)
Trở lại bài toán. Gọi $P$ là giao điểm $DC$ và $(O)$.
Khi đó theo bổ đề $EP$ là tiếp tuyến của $(O)$.
Từ đó dễ dàng dẫn đến đpcm.



#663698 Cho tam giác ABC vẽ về phía ngoài các hình vuông...

Đã gửi bởi Kamii0909 on 03-12-2016 - 14:30 trong Hình học

Xét $Q(B,\frac{-\pi}{2})$ và $Q(C,\frac{-\pi}{2})$ có tích 2 phép quay này là 1 phép đối xứng tâm $Đ_{M}$ do $M$ là trung điểm $DF$.
Theo tính chất tích các phép quay,$M$ là giao của $x,y$ với
$x$ là ảnh của $BC$ qua $Q(B,\frac{-\pi}{4})$
$y$ là ảnh của $CB$ qua $Q(C,\frac{\pi}{4})$
Từ đó $(BM,BC)=(CB,CM)=\frac{\pi}{4}$( mod $\pi$)
Chứng tỏ $\Delta MBC$ vuông cân



#663696 Chứng minh CD vuông góc OE

Đã gửi bởi Kamii0909 on 03-12-2016 - 14:18 trong Hình học

Bài toán vẫn đúng trong trường hợp $A$ không là trung điểm $OD$.
Dễ dàng chứng minh được $\Delta OBD \sim \Delta ECB$
Nên $\frac{BO}{CE}=\frac{BD}{BC}=\frac{CO}{CE}$
Kết hợp với $\widehat{ECO}=\widehat{DBO}$ ta thu được $\Delta COE \sim \Delta BDC$ từ đó dễ dàng có đpcm.



#663371 CHỨNG MINH THẲNG HÀNG

Đã gửi bởi Kamii0909 on 29-11-2016 - 18:04 trong Hình học

Gọi $AI,BI,CI$ cắt $(ABC)$ tại $M,N,P$.
Dễ dàng chứng minh $I$ là trực tâm tam giác $MNP$.
Bổ đề:Cho tam giác $ABC$ trực tâm $H$ nội tiếp $(O)$. Khi đó $\overrightarrow{OH}=\overrightarrow{OA} +\overrightarrow{OB} + \overrightarrow{OC}$.
Áp dụng bổ đề trên với chú ý $I,O$ là tâm ngoại của $MNP$ và $DEF$ và $\overrightarrow{ID} = \frac{r}{R}.\overrightarrow{OM}$ ta dễ có $\overrightarrow{OI}= \frac{R}{r} \overrightarrow{IH}$
Hệ thức trên chứng tỏ $\overline{O,I,H}$.



#662866 Tìm n để trong $S_{n}$ không có số chính phương nào

Đã gửi bởi Kamii0909 on 23-11-2016 - 23:39 trong Số học

Bổ đề là tất nhiên thôi. $\sqrt{a}-\sqrt{b}\geq 1 $ nên giữa $\sqrt{a}$ và $\sqrt{b}$ phải có 1 số nguyên nên ta có đpcm.